click below
click below
Normal Size Small Size show me how
LSAT: Argument
Question Types
Question | Answer |
---|---|
Which one of the following most accurately expresses the conclusion drawn in the argument? | Identify the conclusion |
Which one of the following most logically completes the argument? | Inference |
Laura's statements above, if true, most strongly support which one of the following hypotheses? | Inference |
Which one of the following is most strongly supported by the information above? | Inference |
Which one of the following can most reasonably be concluded on the basis of the information above? | Inference |
Of the following claims, which one can most justifiably be rejected on the basis of the statements above? | Inference |
The educator’s argument proceeds by | Identify the technique |
Paul responds to Sara’s argument using which one of the following argumentative techniques? | Identify the technique |
Which one of the following is a technique of reasoning used in the argument? | Identify the technique |
X responds to Y's argument by doing which one of the following? | Identify the technique |
The claim that … plays which one of the following roles in the argument? | Identify the role |
The claim that … is used in the argument to | Identify the role |
Which one of the following most accurately describes the role played in the argument by the claim that …? | Identify the role |
Which one of the following principles, if valid, most helps to justify the reasoning in the argument above? | Identify the principle |
The journalist’s reasoning most closely conforms to which one of the following principles? | Identify the principle |
The principle stated above, if valid, most helps to justify the reasoning in which one of the following arguments? | Identify the principle |
Which one of the following arguments is most similar in its reasoning to the argument above? | Match the structure |
The pattern of reasoning in the argument above is most similar to that in which one of the following arguments? | Match the structure |
Which one of the following arguments is most closely parallel in its reasoning to the argument above? | Match the structure |
Which one of the following conforms most closely to the principle illustrated above? | Match principle |
The reasoning in the magazine article’s argument is flawed in that the argument | Identify the flaw |
Which one of the following most accurately describes a flaw in the argument’s reasoning? | Identify the flaw |
The reasoning in the argument is most vulnerable to criticism on the grounds that the argument | Identify the flaw |
The reasoning in the argument is flawed in that the argument overlooks the possibility that | Identify the flaw |
The flawed pattern of reasoning in the argument above is most similar to that in which one of the following? | Match flaws |
Which one of the following arguments is most similar in its flawed reasoning to the argument above? | Match flaws |
Which one of the following is an assumption required by the argument? | Necessary assumption |
The argument relies on assuming which one of the following? | Necessary assumption |
Which one of the following is an assumption on which the argument depends? | Necessary assumption |
The conclusion of the argument is strongly supported if which one of the following is assumed? | Sufficient assumption |
The conclusion drawn above follows logically if which one of the following is assumed? | Sufficient assumption |
The conclusion of X's argument can be properly drawn if which one of the following is assumed? | Sufficient assumption |
Which one of the following is an assumption that, if true, would do most to justify X's actions? | Sufficient assumption |
Which one of the following, if true, most strengthens the argument? | Strengthen |
Which one of the following, if true, adds the most support for the conclusion of the argument? | Strengthen |
Each of the following, if true, supports the claim above EXCEPT | Strengthen |
Which one of the following, if true, most seriously weakens the argument? | Weaken |
Which one of the following, if true, is the strongest logical counter that the linguist can make to the philosopher? | Weaken |
Which one of the following, if true, most undermines the claim made above? | Weaken |